PSYC 2131 Final Exam

According to the Article neuroscience in the Courtroom, recent advances in neuroscience have:

raised important questions about free will and personal responsibility

When detectives were asked in a study about their views of profiling, most said they found it:

helpful

According to studies, which form of evidence is most incriminating (likely to lead to conviction)?

Confession

The biggest, most important benefit of having a shadow jury in place, according to the text, is that, because they will actually hear the same testimony as the actual jury, they can provide:

critical feedback that allows attorneys to adjust strategies

_____ can be described as an effort to figure out how the world works, whereas _____ provides a system for meting out just desserts.

science;law

The purpose of exculpatory scenarios is to:

increase probability of admission of guilt through offering a justification for actions.

Marissa is being polygraphed using the guilty knowledge test (GKT). This test focuses on whether Marissa:

knows facts that one would expect that only the criminal would know

According to the text, the term gatekeeper refers to:

the assessment of scientific validity of testimony before allowing it in trial.

According to the text, all of the following shrink the size of the jury pool, EXCEPT:

summon

Jeremy purposely confessed to a crime he did not commit to cover for his wife. This type of confession is referred to as an:

instrumental-voluntary confession.

Get-tough interventions like Scared Straight and boot camps have been found:

not to work and even backfire

The knowledge and skills of developmental psychologists might be relevant for the legal
system in which one of these cases?

When evaluating the state of mind of a teenager who brought guns to school.

The knowledge and skills of clinical psychologists might be relevant for the legal
system in which one of these cases?

When assessing a mentally ill defendant for potential risk of violence

The knowledge and skills of social psychologists might be relevant for the legal system
in which one of these cases?

When studying the dynamics of jury deliberation

Brandeis's brief in Muller v Oregon (1908) was a milestone in the development of the
psychology and law union because it:

opened the door for U.S. courts to use social scientific evidence.

The _____ approach treats laws as a tool that needs to be regularly reexamined and
adjusted, whereas _____ treats laws as evolved to reflect the principles found in nature.

legal realism; "natural law

Legal realists:

believed judges constructed the law through their interpretations of evidence and precedent

Psychologists as evaluators are most likely to be doing research to answer the following question:

does the policy or program work?

Isabella is advocating for a change in the way domestic violence victims are treated by police and the courts. This is an example of the following role played by psychologists in the legal system:

refromer

The roles of psychologists and their influence in the legal system created which positive changes?

judges have to articulate more clearly what their decisions are based on

Differences in goals, methods, and styles of inquiry made the relationship between psychology and law:

difficult but important

Berta is a developmental psychologist. Her knowledge and skills might be most relevant for the legal system in which one of these cases?

when evaluating the state of mind of a teenager who brought guns to school

When psychology and law are discussed as embodying different cultures, the following underlying cultural differences are viewed as most important:

behaviors, values, beliefs

When psychology and law are discussed as embodying different cultures, the following underlying cultural differences between the two fields are considered most important?

goals, methods, and styles of inquiry

Roles that psychologists may play in the legal system include the following:

reformers

On consideration of scientific testimony, gatekeeping in the legal sense includes:

assessing the scientific validity of potential testimony

Ricardo is conducting research to determine the effectiveness of the D.A.R.E. program. This is an example of the following role played by a psychologist in the legal system:

evaluator

The text mentions several impactful ways for scientists influence the legal system. Which of the following is NOT one of them?

lobbying legislators to pass laws requiring the testimony of psychologists at every trial

The text mentions several impactful ways for scientists influence the legal system. What are examples of this?

making scientific findings easy to understand and accessible to the public, participating in continuing education programs that judges and lawyers are required to attend, and disseminating the knowledge gained from research studies by describing them in p

All of the following traits can be defined as vulnerabilities to making a false confession, EXCEPT:

high aggression

Even though a ______ may rule that a confession is inadmissible because it was coerced, such rulings are ____.

judge; rare

Before the 1930s, an interrogator in the United States would be likely to use all of the following actions to elicit confessions, EXCEPT?

imprisoning the suspect's kids

After the publication of the Report on Lawlessness in Law Enforcement in 1931, changes in legislation resulted in the move from _____ physical abuse to ______ forms of abuse

overt; covert

Overt

(adj.) open, not hidden, expressed or revealed in a way that is easily recognized

Covert

(adj.) hidden, disguised, purposefully kept secret; sheltered, secluded; (n.) a sheltered place, a hiding place

In the United States, the following interrogation techniques are legal EXCEPT:

telling the suspect that he will get a reduced sentence if he confesses to the crime

According to research studies described in the text, mock jurors who reported that they disregarded the confession as clearly coerced were ______ likely to convict the defendant.

still more

All of the following statements are true about the use of torture in interrogations, EXCEPT:

there is clear research evidence that torture leads to extracting useful information

After the Miranda v. Arizona (1966) decision, all suspects must be informed of their so-called Miranda rights. These constitutional rights include all the following EXCEPT for the right to:

admit guilt

According to the text, there are several likely reasons that innocent suspects may admit to a crime they know they did not commit. Which one of the following is NOT such a reason?

interrogation techniques are designed to help the suspect sustain self-regulation

A robbery suspect is apprehended and being read his Miranda rights by the police woman arresting him. The suspect is most likely to _____ his Miranda rights.

waive

Police may use a variety of evidence ploys during interrogation. This is possible because:

police can lie during interrogation

During the interrogation of a suspect, openly physical brutality was _____ prior to 1930.

frequently used

Jack was extremely uncomfortable with admitting to a crime against a minor so he kept denying any involvement. The interrogator has been very understanding and nice, and suggested that Jack had committed the crime without realizing the victim was a minor,

certainty of guilt and minimization of culpability

Jillian has suffered from delusional behavior disorder. When brought in for questioning she started out by confessing to a crime that she had taken no part in. The term used for this type of false confession is an:

internalized-voluntary confession

Internalized-voluntary false confessions

are those offered as a means to an end, to achieve some goal. Often, the goal is simply to bring an end to a highly aversive interrogation.
- Possible reason: mental illness

Internalized-coerced false confessions

are the result of a confessor's genuine but false belief that he or she may have actually committed the crime. Coerced false confessions are produced by intense psychological (and occasionally physical) pressure from interrogators, while voluntary false c

instrumental-coerced false confession

occurs when, as a result of a long or intense interrogation, the suspect confesses to a crime that he knows he did not commit. This is the most commonly identified type of false confession in criminal cases. In this type, a suspect becomes convinced that

instrumental-voluntary false confession

occurs when a suspect knowingly implicates himself in a crime he did not commit in an effort to achieve some goal. For example, a member of a criminal gang or organization may voluntarily falsely confess to a crime to protect someone higher up in the orga

Kurt was arrested even though he was innocent. All of the following are likely reasons he waived his Miranda rights, EXCEPT:

this was the first time he heard about the right to remain silent

What are reasons to waive your Miranda rights?

the police explained the Miranda rights in a quick, perfunctory tone, he felt he had nothing to hide, he did not want to seem uncooperative

After an intense interrogation process, Matthew became convinced that he robbed the store since al physical evidence pointed at him. This form of false confession is called an:

internalized-coerced confession

Lying can be best described as:

multifaceted

According to the text, the "____________ Hypothesis" posits that human intelligence was accelerated by the need for humans to develop social skills related to manipulation and deception.

Machiavellian

Based on research studies of people's abilities in lie detection, the following statement is true:

police detectives are no better at detecting lies than college students

the comparison question polygraph technique utilizes comparison questions that are supposed to make a person uncomfortable (so called known lie questions). Which one of the following questions is NOT a good comparison question for CQT purposes?

Have you ever tried a food you did not like?

What are examples of good CQT comparison questions?

have you ever lied to get out of trouble

Jared was questioning a suspect using the Reid technique, and after a long and exhausting interrogation, the suspect falsely confessed to the crime just to put an end to the interrogation. When hearing this confession, Jared felt even more confident that

confirmation bias

attribution bias

the tendency to attribute one's own negative behavior to external causes and one's positive actions to internal states

When jurors are presented with polygraph evidence they:

take such evidence seriously

The technique that uses systematic analysis of written statements to assess the credibility of accounts related to a particular event is called:

criteria-based content analysis (CBCA)

The rates of accuracy of the polygraph have been calculated using controlled lab studies and mock crimes. These rates are likely to be somewhat inflated for the following reasons, EXCEPT:

the polygraph machines used in these studies are outdated

There were several problems with standard polygraph techniques like CQT. Which of the following is NOT one these problems?

polygraph examiners believe in the importance of their mission and skills

Techniques such as biting one's tongue, pressing one's toes to the floor, or counting backwards are all examples of:

countermeasures to polygraph testing

The following behavioral clues are a reliable indicator that the person is lying:
- cross legs or arms
- fidgeting or stuttering
- averting gaze or avoiding contact

none of the above

The ______ shows a video image of the brain in action

fMRI

According to the text, the actual level of accuracy of polygraph techniques is likely:

between 50% and 90%

The basic theory behind the polygraph is that lying:

causes physiological arousal

Paula has been playing with her fingers and rings during the interrogation and tried to avoid direct eye contact with her interrogator. Most people would mistakenly conclude that she is hiding something. This erroneous belief is called:

liar's sterotype

false positive

an innocent person is misclassified as guilty after telling the truth

false negative

a guilty person is misclassified as innocent after lying

true positive

a guilty person is correctly classified as guilty after lying

true negative

an innocent person is correctly classified as innocent after telling the truth.

If a person is lying, the polygraph can pick up the following changes in his or her physiological functions, EXCEPT:

brain acitivity

If a person is lying, the polygraph can pick up the following changes in his or her physiological functions...

breathing, perspiration, heart rate

The guilty knowledge test (GKT) uses a multiple-choice question format. When a suspect provides the answers, ______ is usually _____.

the first choice; thrown out

According to "Lie Spotting" author Pamela Myer's talk

we all lie frequently for various reasons, some lies are harmless, lying is a cooperative act

During the trial, each side is trying to win the case. However, the goal of the legal system overall is to:

achieve justice

The primary goal of psychological science is to:

provide accurate explanations of human behavior

According to the text, the goal of psychology is to ____, whereas the goal of the legal system is to _____.

emphasize the characteristics of groups; emphasize individual cases

Precedents can best be described as:

legal cases that have established a rule that are then used to decide future cases
with similar issues

The basis of the adversarial system in law is that:

truth will emerge as a result of contest between opposing sides

Differences in goals, methods, and styles of inquiry made the relationship between
psychology and law:

difficult but important

Sylvester is hired by the defense to help select jurors who would be less likely to convict. This is an example of the following role played by psychologists in the legal system:

advisor

Trial consultants may be hired to perform the following duties:

preparing witnesses and shaping trial strategy

Sam is a cognitive psychologist. What aspect of the legal process might he provide assistance with to the court?

Clarifying whether jurors understand the instructions for deliberating a verdict

Studies evaluating the effectiveness of the D.A.R.E. program found that:

the program is not effective in reducing drug use

Ricardo is conducting research on the effects of solitary confinement on mental health. This is an example of the following role played by a psychologist in the legal system:

evaluator

For a reasonable evaluation of scientific validity of potential expert testimony, one needs to have sufficient understanding of:

research methods

The Daubert trilogy of cases impacts trial courts by:

putting the burden of decisions about allowing expert testimony on trial judges

The ways Daubert trilogy affected the legal system includes all of the following
EXCEPT:

superior courts are more likely to question trial judges' decisions about allowing expert testimony

A psychologist testifying in court feels it is his responsibility to correctly and clearly present scientific findings, even if this may lead to an unfair verdict by the jury. According to Saks (1990), such an expert fulfills the role of:

conduit-educator

Amicus curiae briefs:

provide a way to influence the court when expert testimony is not allowed

According to the textbook, when writing amicus curiae briefs, it may be hard for psychologists to balance the goals of:

science translation and advocacy

The roles of psychologists and their influence in the legal system created which positive
changes?

Judges have to articulate more clearly what their decisions are based on.

A fingerprint examiner's decision about a match may be influenced by whether he is told the suspect has confessed to the crime. This tendency to seek out information that supports one's beliefs is known as:

confirmation bias

Fingerprint evidence is ____ for jurors to understand because unlike DNA analysis information, it does not involve______

easier; probabilities

Bullet-matching results

are inconsistent and unproven, even after they have been used in thousands of trials

Validity refers to

whether the measure is actually measuring what it is supposed to measure

A false positive means that a(n):

expert declares a match when there is no match in reality

Biological evidence may include:

blood, saliva, semen, and skin cells

Forensic identification is less likely to result in inconclusive findings when a _____ left at the crime scene is ______>

tire thread; clear

All of the following problems plague forensic science, EXCEPT:

each scientific study undergoes extensive scrutiny by peer reviewers

All of the following problems plague forensic science

most examiners lack advanced graduate training in scientific methods, forensic science has been marked by secrecy and fear of negative findings, and financial ties and conflict of interest are common in forensic science

Pierre has received a piece of pizza crust with DNA evidence from the crime scene to analyze and compare it with the CODIS database for possible matches. However, because the piece of pizza crust was mistakenly stores in a plastic bag instead of a paper b

Pierre has to

Kelly works as a fingerprint examiner. A police detective showed her some gory photos of the crime scene and told her that the suspect in custody is a likely perpetrator of the murder. Kelly is more likely to find a match between the latent prints and the

contextual bias

Forensic identification can be defined as the process of linking:

physical evidence to a particular individual

Face identification technology has become much more accurate in recent years, but still has its limitations. For example, the face cognition security on the iPhone X can be easily fooled by:

a family member's face

Researcher Daniel Kahneman gave the following problem to a variety of people:

80%; 40%

Like with any other technique, what would be required to demonstrate the usefulness of profiling is:

systematic research

The characteristics of serial killers seem to have ____ though there are _____ common to all serial killers.

some recurring patterns; no characteristics

Many serial killers suffer from ____ that impairs their ____

brain injury; rational thinking

According to the text, courts have been reluctant to allow expert testimony based on psychological autopsies. One exception to this rule is a routine use of psychological autopsies in cases where:

the will is contested based on whether the deceased person was legally competent when the will was written

In geographic profiling, an anchor point refers to:

the location from where attacks might be launched

As a rule, ____ approaches have been found to be superior to ______ approaches.

statistical; intuitive

The NASH system is utilized to classify the:

causes of death

Profiles are created primarily for the purpose of:

developing some leads in the ongoing investigation

One of the most serious ____ of profiles is that _____ profiles may direct the police to look at the wrong suspects.

problems; misleading

Victims in a string murder cases were all mutilated in a specific way, which makes profilers think of this feature as a(n) _______ that may reveal something about the perpetrator's personality.

signature

Correll and associates asked various groups to participate in a First Person Shooter simulation. In the simulation, participants saw unarmed and armed targets, and were suppposed to shoot the armed targets and not shoot the unarmed targets. Correll's resu

police officers had a slight bias towards shooting both white and black targets, college undergraduates had a bias toward shooting black targets but not white targets, community members showed a bias toward shooting both black and white targets, but the b

Vera Renczi is an example of this type of female serial killer:

Black Widow

Aileen Wuornos is an example of the type of female serial killer known as the _____ _______

sexual predator

One reason why adult hearsay witness testimony may be perceived differently from that of a child, is that:

adults are more confident in their testimony

There are good reasons why most states allow hearsay testimony when a child is the alleged victim. Which of the following is NOT such a reason?
- to avoid the traumatic experience for the victim of facing the alleged abuser
- to spare young children the i

to shield jurors from the biasing impact of young children's cuteness

According to the text, the use of anatomically detailed dolls when interviewing children under the age of 6 is likely to ____ the number of false allegations of sexual abuse

increase

The use of a closed-circuit television (CCTV) is another way that children can testify in a case. In the case of Maryland v. Craig, the U.S Supreme Court decided that the ______ the defendant's rights to confront his or her accuser.

importance of protecting the child victim outweighs

Hearsay testimony is generally _____ in court but most states do allow an exception to the hearsay rule for ______ victims.

inadmissible; child

Liam is a 4-year-old who is being questioned by his mom about his physical contact with a guest visitor to Liam's day care center. Even though the visitor never touched Liam, the mother keeps asking which part of Liam's body the visitor touched first. Lia

tell mom what she wants to hear even if he has to make it up

Research exploting the effects of repetitive questioning on preschool-aged children's responses found that:

most children changed their answers to incorporate the suggested falsehoods

According to the text, the following statements about child sexual abuse are true, EXCEPT?
- the true prevalence of child sexual abused is difficult to measure exactly
- the true prevalence of child sexual abuse could be higher than the current best estim

about equal percentages of boys and girls experience some form of unwanted sexual touching

According to the text, all of the following ways ease the child testimony burden have been suggested. EXCEPT:
- bringing a support person to court
- bringing a support animal to court
- familiarizing the child with courtroom procedures and personnel

having siblings administer the cross-examination of the child

In a series of experiments replicating Loftus research, Hyman and his colleagues set out to create false memories of fairly unusual past events. The results of their studies show that again, about ____ of the participants had developed false recollections

a quarter

Research by Gail Goodman and her colleagues (1998( examined the use of a closed-circuit television (CCTV) for child testimony. The result shows that the use of a CCTV ______ the emotional distress in children _____ the accuracy of their testimony

decreases; and improves

Cross-race effect has been shown to maintain consistent strength:

for people of different ages

According to the text, the cross-race effect is present in babies as young as ____ old, and ____ from childhood to adulthood.

9 months; is consistently stron

Children's memories are likely to be impacted by all of the following, EXCEPT:

their lack of understanding of the court precedings

_____ is the part of the memory process that involves accessing and pulling out stored information at a later time

retreival

The maintenance of information over time within the brain is referred to as:

storage

Hypnosis was once considered a promising technique for refreshing witnesses memories. However, it turned out to have some serious downsides. these downsides include all of the following EXCEPT:
- hypnosis can elicit false memories
- hypnosis can boost the

the most vivid memories can not be suppressed by hypnosis

According to research by Penrod & Cutler (1999), estimates of time during the course of a stressful event are ____ the actual length of the event.

three to four times

The weapon focus effect means that the eye witnesses ability to remember the attacker is _____ if the attacker is holding a ___.

impaired gun or knife

Sabrina works as a bank teller. When recalling the event, she is likely to estimate that 30-second time period to have lasted about:

2 minutes

When describing memory, _____ is the first component required in the process necessary for retention.

encoding

The reassignment of a face that is familiar from another context to the scene of a crime is referred to as:

unconscious transference

When Jennifer Thompson saw her real rapist, Bobby Poole at Ronald Cotton's second trial, she

swore that she'd never seen Bobby Poole before, and that Ronald Cotton raped her

According to a recent experiment conducted by wells and colleagues, real eyewitnesses to real crimes, who actually choose someone out of a lineup, choose the wrong person about ____ of the time, even when "best practices" in lineup procedures are used:

30%

bona fide doubt may also be referred to as

reasonable doubt

Psychological tests related specifically to legal issues, for example, for the evaluation of competence to stand trial, are referred to as:

forensic assessment instruments

Even if the accused is deemed incompetent to stand trial, he or she ____ to become incompetent, in the legal sense. According to the text, successful restoration of competency to stand trial happens for ____ of incompetent defendants.

may receive treatment; the majority

All of the following are advantages of performing competency evaluations on an outpatient basis, EXCEPT:
- the defendant does not need to b incarcerated during outpatient evaluations
- they are less intrusive than inpatient evaluations
- they are cheaper

they can be easily manipulated due to an uncontrolled environment

It is important to establish that the defendant is competent to plead guilty because the guilty plea means giving up the following constitutional rights, EXCEPT for the right to:
- remain silent
- confront one's accusers
- appeal

get a fair sentence

Competence to stand trial is most likely to be compromised if the accused:

is in a severe psychotic state

Which legal actors are ethically bound to assure that a defendant is competent to stand trial?

the defense attorney judge and prosecuting attorney

What is a primary advantage of providing an inpatient rather than an outpatient evaluation of competency?

it provides multiple opportunities to observe the defendant's behavior over time

Competency to stand trial (CST) refers to the psychological state of the accused:

at the time of trial

To commit someone to a mental institution for an indefinite period of time, an individual must be deemed wither ____ or ____.

unable to provide for his or her basic needs; imminently dangerous to self or others

As defined in the text, malingering means that an individual is:

intentionally faking a mental illness or disability

In the U.S. legal system, the right to waive an attorney implies that the defendant:

is competent to defend himself in court

Another example of mistaken public views on what happens with NGRI defendants is a common idea that they get _____ sentences than non-NGRI defendants. In reality, people who are deemed legally insane often spend ___ time in custody than non-NGFRI criminal

shorter; more

Whereas competence refers to the defendant's state of mind _____, insanity concerns his or her state of mind _____.

at trial; during the commission of the crime

When it comes to the issue of insanity defense, four states (Montana,Utah, Kansas, and Idaho) have:

entirely abolished the insanity defense

The idea of insanity defense is based on the principle that:

people who commit crimes without full awareness should not be held fully responsible for their behavior

Most of what people believe about the insanity defense is mistaken. For example, the proportion of felony cases where the defendant uses the insanity defense is less than ____; and when the insanity defense is used, it is successful in about _____ of thos

1%; 25%

One of the most famous examples of malingering involved the serial killer Kenneth Bianchi who was pretending to be _____ and faked _______.

hypnotized; multiple personalities

An increasing use of neuroscientific evidence like brain scans in the courtroom led to the creation of a new interdisciplinary field called _____. A fundamental conflict embedded in this interdisciplinary field concerns the issue of ______.

neurolaw; free will

Due to the diassastification with both the M'Naughten rule and the Durham standard. It offered a two-prong approach for insanity; one related to the _____ prong, or an ability to appreciate the wrongfulness of one's conduct, and the other related to the _

cognitive; volitional

According to the text, the _____ refers to a situation where the impulse is so overwhelming that the perpetrator would have committed the act even if a law enforcer stood beside him or her at the time of the crime.

policeman at the elbow test

If prosecutors contest insanity, jurors are typically not informed that the defendant would not go free if found NGRI. A study cited in the text has found that telling jurors what happens to a defendant found NGRI:

leads to more NGRI verdicts

Which of the following statements is TRUE?

Kenneth Parks was found not guilty because because of sleepwalking when he drove 14 miles and beat his mother-in-law to death

Experts believe that sleep disorders are increasing in our society because of ______.

an increase in sleep deprivation in our culture

At the ______ level of responsibility, we are held responsible for consequences that we personally caused, IF a reasonable person could have predicted the consequences of their action.

forseeability

Heider's _______ level of responsibility is relevant to the legal concepts of self defense and insanity

justification

The three phases of abuse, according to Lenore Walker include all of the following, EXCEPT:

release

A study cited in the text found that the most likely combination to sway jurors toward more lenient sentencing for a battered woman who killed her abusive husband is expert testimony on _____ combined with a(n) ______ instruction.

BWS; judicial nullification

Men are _____ women to seriously injure or kill their intimate partners

several times more likely than

Rape trauma syndrome (RTS) has been:

just as controversial and potentially flawed as BWS

Hypervigilance can be described as:

heightened attractiveness to the abuser's subtle behaviors

The tension building phase of the cycle of violence, as hypothesized by Lenore Walker, is often represented by _____, whereas contrition phase often includes ______ expressed by the abuser.

arguing, remorse

The term rape trauma syndrome (RTS) can best be described as a (n):

cluster of symptoms identified in a sample of rape victims

Studies have found that women who used violence against their abusers were likely to be found ______ in a jury trial, and expert testimony on BWS was likely to ________

guilty; make little difference

Multiple studies have found that women who have experienced _____ trauma in childhood or adolescence are _____ to face another such attack or abusive relationship in adulthood.

sexual; several times more likely

The battered women syndrome is intended to explain all of the following, EXCEPT:
- Why a woman abused by her partner is likely to become more isolated and dependent on her abuser.
- Why a battered woman is likely to believe she has brought the abuse upon

Why a woman trying to leave an abusive relationship is likely to be killed by her abuser

Francine Hughes was tried for murdering her abusing husband, and was found

not guilty by reason of temporary insanity

According to the investment model, Susan would be most likely to LEAVE her abusive marriage if

Susan's satisfaction level with other aspects of her marriage is relatively low.

A judge _____ accept(s) the challenges put forth by attorneys regarding jury selection.

sometimes

Voir dire is all of following EXCEPT:
- an opportunity for attorneys and the judges to question potential jurors
- a sort of pretrial interview hold in open court
- the final stage in the selection process

a way to exclude attorneys from influencing the trial outcomes

According to the Jury Selection and Service Act of 1968, the voter registration list is the "__________ source" of eligible jurors.

primary

According to the text, all of the following shrink size of the jury pool, EXCEPT:
- exceptions
- no-shows

summons

In theory, the number of challenges for cause is ______, while peremptory challenges _____.

unlimited; are capped at a small number

If juries systematically exclude ______, the legal system _____.

cognizable groups; loses its legiitimacy

As opposed to early juries at the dawn of the United Staes, modern-day juries consist of people who:

do not have a prior knowledge of the defendant.

April displays conventional values, with relatively inflexible viewpoints, generally identifying with authority figures and finding weakness to be intolerable. One might consider her personality to be that of _____, which one of the jurors characteristics

authoritarianism, harsher

John was summoned for jury duty According to the text, what he used is called the _____ plea.

undue hardship or extreme inconvenience

It would appear as though there are certain personality traits or tendencies that have some bearing on jury verdicts. However, these traits only play a role in the jury decision if the:

evidence presented by the prosecution is weak or ambiguous

Research studies cited in the text show that trial consultants on average _____ affect the outcome of the trial, whereas some especially experienced lawyers ____.

do not; substantially tilt the odds of winning the trial

Some legal scholars have been advocating for a system of professional jurors who can understand the complex evidence presented in court and know the law related to each case. Which one of the following is NOT among these problems?

It is not clear what course of studies professional jurors should complete to prepare for their jobs.

According to the text, juries that require unanimity to reach a decision, deliberate _____ and are also more likely to ____ than majority-rule juries.

longer; hang

During the jury deliberation process, two distinct styles are evident. Some juries adopt a(n) _____ style, while other juries choose a(n) ______ style.

verdict driven; evidence driven

The lack of clear evidence often leads jurors to consider other factors such as pretrial publicity and prior beliefs. This phenomenon is known as the ______ hypothesis.

liberation

when a judge dictates that certain evidence is not be ignored, jurors often feel that their freedom to choose to consider all evidence all threatened. Reactance theory suggests that this belief often motivates the jurors to consider ______ evidence to a(n

inadmissible; greater

typically, jurors are provided with instructions _______. Studies show that preinstructions given to a jury before the beginning of a trial help jurors _____.

only after the trial is over; evaluate evidence differently

Should judges or juries decide the verdict and sentencing? Based on the findings of these studies, all of the following statements are correct, EXCEPT:
- both judges and jurors are equally likely to be influenced by biasing information
- both judges and j

judges correctly identify the bases for their sentencing decisions

During George Zimmerman's trial for killing Trayvon Martin, all of the following circumstances likely contributed to the jury's verdict of "not guilty," EXCEPT:
- the jury had no African American jurors
- Voice analysis experts were not allowed to testify

hoodies are a prohibited item of clothing in gated communities

______ occurs when group pressure results in holdout jurors changing their votes, even if their opinions of the case did not change

normative influence

When an expert witness is presenting a testimony that is ______, the strength of expert's credentials is ______.

very complex; especially important

Research evidence suggests that, when judges and juries disagree, all of the following is true, EXCEPT:
- the jurors understanding of complex evidence is just as good or maybe even better than the judges understanding
- juries are more lenient than judges

such disagreements between judges and juries never happen

The syndrome coined by Richard Garner that describes one parent attempting to induce fear of the parent in the child is called ______ syndrome

parental alienation

Some scholars have suggested that the child must spend the equivalent of two overnights a weeks for the custody to be considered shared. This type of custody is also known as:

joint physical custody

Studies have identified several factors that help children's better adjustment after divorce. Which one is NOT such a factor?
- minimal conflict between parents
- parenting that involves high support and high control
- close relationships between the chil

more permissive parenting on the part of both parents

The goal of the approximation rule is to:

maintain as much stability for the child as possible

Psychologists become involved in cases that include child custody arrangements because psychological evaluation of one or both of the parents is wither required or recommended. In parental evaluations, individuals are more likely to ______ psychological p

hide; faking good

Kathryn's parents recently divorced. During the hearing for custody, it was determined that her mother would have legal and physical custody. This type of custody is known as

sole custody

All of the following statements are true about child custody decisions, EXCEPT:
- definitions of what constitutes shared physical custody and not uniform
- there is no single custody arrangement that is best in all situations
- most custody arrangements a

the most common custody arrangement is joint custody

All of the following are noted in the text as benefits of joint legal custody, EXCEPT:
- both parents remain closely involved in raising the child
- the non-custodial parent is more likely to pay child support on a regular basis
- family cohesiveness is i

parents share their time with the child equally

Research that follows subjects and collects data over a longer period of time is referred to as:

longitudinal

In recent decades, the instances of joint legal custody have been:

increasing

In custody decisions, the approximation rule states that the child should:

remain in close placement that is true to the caretaking relationship that existed prior to the divorce

The burden of proof for civil commitment is:

by clear and convincing evidence

Only three states have been responsible for over half of the total executions in the United States since 1976:

Texas, Oklahoma, and Virginia

Static, or unchanging markers that are used for predicting future violent behaviors are also called:

historical markers

In many death penalty cases, the sentence is decided based on the _____ standard, which is a probability that the defendant would commit an act of criminal violence and thus presents a continuous threat to society

future dangerousness

The early studies of recidivism that tracked people released from confinement found that:

the vast majority of people deemed violent did not reoffend

____ was the first person to be subjected to the ____ laws (SVP laws).

Leroy Hendricks; sexually violent predator civil commitment

involuntary civil commitment is based on what:

A person might do in the future

Predictions made by psychological experts regarding potential future violence are almost uniformly _____ by the courts.

upheld`

Violent behaviors are only slightly more likely among people with severe mental illnrss. According to the text, a much more important factor is ______, which _____ violent behavior even among mentally healthy people.

substance abuse; increases

When evaluating risk assessment evidence, it has been found that jurors usually find _____ predictions more convincing. Research shows that ______ predictions are more accurate

clinical; actuarial

Researchers have determined three broad categories of predictors of risk for future violent behavior:

historical, dynamic, and risk management markers

_____ states include sexual orientation in hate-crime legislation.

19

Hate speech" can lead to increases in hate crime because threatening language _____

activates the amygdala and makes it harder for people to dial down their emotions

According to FBI statistics, hate crimes in the United States

increased dramatically from 2016 to 2017

In 2017, most religiously-motivated hate crimes targeted people of the ____ faith.

Jewish

Which one of the following innocent people was NOT executed?

Kirk Bloodsworth

In Roper v. Simmons (2005), the Supreme Court abolished the death penalty for _____ because psychological research shows that their key psychological capacities are _____.

juveniles; not yet fully developed

The purpose of the death penalty is seen as deterrence. However, many studies in the United States, as well as in other countries, have found _____ evidence to support the idea that capital punishment deters murderers. Moreover, there is research showing

no clear; increases

In protest of the death penalty, many countries:

help their citizens facing executions in the United States

Many countries refuse to ____ in protest of the death penalty.

extradite death penalty eligible criminals to the United States

Mitigating factors in capital cases are characteristics that make:

executions less appropriate as punishment

In the reforms set forth in Gregg v. Georgia (1976), defendants on trial for capital murder must be tried by juries in:

two phases

Following Gregg v. Georgia and its companion cases, all death sentences are:

reviewed by state supreme courts

In Gregg v. Georgia (1976), and in companion cases, the Supreme Court set up a series of reforms. These reforms are referred to as the:

guided discretion statues

Interestingly, there appears to be a tendency for an increase in murders for a short while following an execution. One possible explanation is that executions may desensitize people for killing. This phenomenon is referred to as the _____ effect.

brutalization

A study tracked jurors emotions during a capital penalty trial. Researchers found that the more intense a juror's ____, the _____ likely that juror was to vote for death.

anger; more

In most countries with the death penalty other than the United States, the common methods of execution include all of the following, EXCEPT:
- beheading
- hanging
- shooting

lethal injection

Which of these is most commonly associated with convictions of the innocent?

mistaken identification

In the 1960s-1970s, all of the following tendencies were developing, EXCEPT:
- psychologists were becoming more eager to apply research to law-related matters
- the legal system was becoming more receptive towards social scientific research
- there was en

there was growing pessimism about the alliance of psychology and law

Jasmine is a social psychologists. Her knowledge and skills might be most relevant for the legal system in which one of these cases?

When studying how the type of interrogations is related to the likelihood of false confessions

At the end of the 19th century, Justice Oliver Wendell Holmes famously said: "Law is the government of the living by the dead." His saying reflects the currently relevant principle that:

legal rulings in cases decided a long time ago may still apply to current cases

Research by Kassin and his colleagues has shown that even when _____ had no problem recognizing that a confession was coerced, they still voted _____ more often.

jurors; guilty

Michelle is in custody and being interrogated. The behavior of the police officers is an example of:

using a good cop-bad cop approach

According to the text, research on jurors' perceptions of confessions show that:

it is very difficult for jurors to discount a confession, even when told it is a lie

Hugo is interrogated alone and told repeatedly that his guilt is corroborated by both physical evidence and eyewitness accounts. This approach most likely embodies the following strategy used by police during Hugo's interrogation:

social isolation and certainty of guilt

Which one of the following patterns of thinking demonstrates the fundamental attribution error, or the tendency to attribute other people's behavior to dispositional causes?

If the suspect were innocent, he would not have confessed to the crime he did not commit.

According to the text, the current state of the criminal justice system in the United States makes all of the following very likely to happen, EXCEPT:

juries can often determine if the confession resulted from psychological coercion

The Reid technique is _____ used by police in the United States when questioning suspects. Establishing a sense of loss of control as part of this technique makes suspects feel ____.

often; vulnerable

The guilty knowledge test (GKT) is one of the most promising polygraph techniques to date. However, there are some limitations to its usefulness. Which of the following is one of the min limitations of GKT?

A sufficient number of crime facts uniquely known by only the criminal is not available

Training people to detect lies ____ their ability _____ their confidence in their ability to discern lies.

does not improve; but raises

According to the text, when police use the polygraph, it is usually done for the purpose of:

coercing the suspect to confess

The polygraph may be used as a ____ tactic to evoke ___.

coercive; a confession

Among the techniques looking at brain activity patterns, ______ is better able to locate the precise area of the brain activity, whereas _____ is better able to determine the timing of the activity.

fMRI; EEG

Elena was suspected of committing a fraud. Investigators asked her to tell her story in reverse. This promising lie-detection technique is based on the idea that narrating backwards:

increases cognitive load

In the movie "Forgotten Sins," Matthew's first "confession" occurs

under interrogation with a police psychologist present

Legal realism, as expressed by Karl Llewellyn in the 1920s-1930s, embodied all of the following viewed, EXCEPT:

law should be judged independent of of its effects

Courts have been receptive to clinical psychologists than to to other types of psychologists. The most probable reason is that, like lawyers, clinicians:

are oriented toward the individual case

The knowledge and skills of developmental psychologists might be most relevant for the legal system in which one of these cases?

When evaluating which custody arrangement will most benefit the child

The knowledge and skills of clinical psychologists might be most relevant for the legal system in which one of these cases?

When assessing a mentally ill defendant for potential risk of violence

In the United Kingdom, the Police and Criminal Evidence (PACE) Act was enacted to make it illegal to trick suspects or to lie for the purpose of coercing suspects to confess. Studies show that _____ of pressure tactics and trickery, suspects are _____ lik

despite the decline; just as

According to the textbook, police prefer a confession because:

confessions save time

The comparison question test (CQT) posits that guilty individuals reacts more strongly to ____ questions, whereas innocent individuals react more strongly to ____ questions.

relevant; comparison

The federal Polygraph Protection Act of 1988 prohibited the use of polygraph by:

most private employee for the purpose of hiring for a job.

According to the text, all of the following statements about false confessions are true, EXCEPT:

studies prove that false confessions are extremely rare

According to the text, even though fMRI is a promising technique, it has multiple serious limitations. Which of the following is not one of these limitations?

The fMRI machine is highly portable

Lying is considered an adaptive behavior, from a(n) _____ point of view.

evolutionary

Which of the following statements is NOT true?

Most people never tell lies.

According to the text, the most prevalent use of the polygraph in the United States is:

for national security purposes

The ability of humans to maintain relatively harmonious social relationships is facilitated by our capacity to ______ the truth.

bend and reshape

According to research completed by Bond and DePaulo (2006), people's ability to distinguish lies has a _____ above-chance rate of accuracy, and efforts to improve people's ability to discern lies have ____.

barely; not been very successful

_____ measures almost imperceptible changes in heat of human face.

High-definition infrared thermal imaging

Studied show that those police officers who have passes through interrogation training:

feel more confident in their judgments of lie detection even when wrong.

According to "Lie Spotting" author Pamela Myers' talk, the best cue to a "fake" smile is

the eyes

When researchers assessed police detectives' ability to detect lies, they found the police to be ____ likely to correctly assess the truthfulness of videotaped statement compared with college students.

less

Desiree has been interrogation room for a while. She is not sure how long the interrogation can last or even if she can ask for food and water. She gets increasingly uncomfortable and worried. This situation illustrates which one of the four basic influen

loss of control

A psychologist testifying in court feels it is his responsibility to correctly and clearly present scientific findings, even if this may lead to an unfair verdict by the jury. According to Saks (1990), such an expert fulfills the role of:

conduit-educator

During the trial, attorneys make opening statements and closing arguments for the purpose of:

advancing their version of the evidence

When the police utilize a good cop-bad approach, the suspect is likely to:

confess to the good cop when alone with him

The text identifies three fundamental problems with forensic identification. Which of the following is NOT among these problems?

Forensic Identification is too often conducted at the crime scene than in a lab.

Biometrics is the:

identification based on measurable anatomical traits.

______ measures almost imperceptible changes in the heat of the human face.

high-definition infrared thermal imaging

The comparison question test (CQT) posits that guilty individuals react more strongly to _____ questions, whereas innocent individuals react more strongly to ______ questions.

relevant; comparison

Which of the following statements correctly reflects the current legal status of the polygraph?

Almost half of the states have banned the use of the polygraph results in court

When researchers assessed police detectives ability to detect lies, they found the police to be ____ likely to correctly assess the truthfulness of videotaped statements compared with college students.

less

According to Bella Depaulo's scientific review article on lie spotting.....

- nonverbal signs of

According to the text, confirmation bias is one's tendency to:

seek out evidence that supports one's preexisting beliefs

According to the text, EEG reads neural impulses ______, whereas fMRI reads brain activity _____.

continuously; every 1.5 seconds

A survey was administered to high school seniors in Anytown. According to the survey results, fewer than 0.5% of the students drove drunk in the previous 6 months. A month later, the survey was repeated and it again indicated that fewer than 0.5% of Anyto

high test-retest reliability

All of the following problems lead to fingerprint evidence being imperfect and prone to error, EXCEPT:

If the number of matching ridge characteristics reaches 16, the two prints match.

A survey was administered to high school seniors in Anytown. According to the survey results, fewer than 5 of them drove drunk in the previous 6 months. However, Anytown police records indicate that more than 15 Anytown seniors were arrested for driving d

low measurement validity

_______ the other, well-established forms of forensic identification, DNA identification was subjected to ____ scrutiny to establish its scientific validity and relevance.

Unlike; increased

According to the text, it is not easy to calculate the false positive rate for most types of trace evidence because:

forensic examiners do not want to participate in studies of error rates.

At each locus of the DNA strand, a person has two genetic characteristics, or ______: one inherited _______.

alleles; from each parent

DNA can be extracted from _____ evidence.

biological

Face identification technology has become much more accurate in recent years, but still has its limitations. For example, the face recognition security on the iPhone X can be easily fooled by:

a family member's face

When profilers create a profile of the criminal based on the information about the crime, one could say they are constructing a:

tentative description

Serial killers tend to select victims:

of a particular type.

One of the characteristics thought to be indicative of childhood maladjustment has been identified among quite a few serial killers. It is:

cruelty to animals.

In a study by Pinizzotto and Finkel (1990) that has compared the accuracy of profiles developed by trained profilers with those of students, it was found that _____ created more accurate profiles ______.

profilers; but only for sex offfenses

Most serial killers are _____ males of _____ intelligence.

white; average

Rational thinking, in serial killers, is often:

impaired

If the imprint came from a popular type of Adidas sneaker, it reflects _____; if the imprint has a unique pattern of wear and tear, it reflects_______.

group characteristics; individual characteristics

DNA can be used more successfully to infer some individual characteristics rather than others because environmental factors and people's behavior influence certain traits more than other ones. Which one of the following characteristics is the easiest to f

race

Ebony works as a fingerprint examiner. In a blind test, she asked to determine if the two fingerprints match. She does not know that it is the same pair of fingerprints she has evaluated a few months before. To compare her previous decision with her curre

test-retest reliability

Nicholas was brought in for questioning as a result of detailed police work, including fingerprints. He was "excluded" as a suspect. This means that the fingerprints found at the crime scene were:

found to be from someone else

Among the types of questionable forensic evidence that has been used in courts for decades, the text mentions the chemical composition of the bullets and handwriting analysis. What is the reason these are not valid techniques?

the error rate is too high to make the techniques useful

Refers refers to:

the consistency of a measure or observation

Following Jackson v. Indiana (1972), it is generally accepted that an individual found incompetent to stand trial should be ____ for between 4 and 18 months, and then ______.

treated to restore competency; reevaluated

It is very important to establish that the defendant is competent to plead guilty because the guilty plea means giving up the following constitutional rights, EXCEPT for the right to:

get a fair sentence

Even if the accused is deemed incompetent to stand trial, he or she ______ to become competent, in the legal sense. According to the text, successful restoration of competency to stand happens for ______ of incompetent defendants.

may receive treatment; the majority

The legal concept of whether an individual has the capacity to perform necessary personal or legal functions is known as:

competence

According to the text, all 50 states allow juveniles of a certain age to be _____ when charged with serious crimes.

transferred to adult court

In the case of Jared Loughner described at the start of the chapter, the judge ordered Loughner's competency evaluation because the defendant:

had strange beliefs about how the legal system works.

If a defendant is found incompetent to be executed, proceedings are initiated to _____ so that he or she can be ____.

restore him or her to competency; executed

Phillip is standing trial for robbery. Before he goes to trial, it is important to assess whether he:

is competent to stand trial

Eric was scheduled for trial as a result of a felony offense. It was soon discovered that he was certain that the light fixtures throughout his city were purposely set there by aliens trying to learn how to "fit in." He was found incompetent to stand tria

his delusions hampered his ability to assist in his own defense since he believed that his attorney was a spying alien

Volitional capacity refers to the defendant's

inability to control his or her behavior

According to the text, the ____ refers to a situation where the impulse is so overwhelming that the perpetrator would have committed the act even if a law enforcer stood beside him or her at the time of the crime.

policeman at the elbow test

Developments related to the insanity defense can be traced back:

several centuries

Legal definitions of insanity are crafted by:

legislators and judges

The M'Naghten rule lists several components to meet the standard of legal insanity. One of the components is that, at the moment of the crime, the accused must have:

been laboring under a defect of reason

Studies have analyzed wrongful conviction cases that were later overturned because DNA testing proved the wrongfully convicted person innocent. Of the following major causes of wrongful convictions, which one is found to be the most prevalent?

Eyewitness misidentification

The tendency of people to be better able to recognize faces of those within their own racial group than those outside their racial group is referred to as the _____ effect.

cross-race

The weapon focus effect means that the eyewitnesses' ability to remember the attacker is _____ if the attacker is holding a _______.

impaired; gun or knife

if prosecutors contest insanity, jurors are typically not informed that the defendant would not go free if found NGRI. A study cited in the text has found that telling jurors what happens to a defendant found NGRI:

leads to more NGRI veridcts

The Insanity Defense reform Act (IDRA) of 1984 has elimated the _____ from the definition of insanity and prohibited from providing ______ testimony.

volitional capacity; the ultimate issue

Experts believe that sleep disorders are increasing in our society because of _____.

an increase in sleep deprivation in our culture

Which of the following statements is FALSE?

only about 5% of all children sleepwalk at some point

Heider's level of ______ seems to be most important in allocating punishments.

forseeability

Because BWS and RTS are often difficult for the _____ to integrate into a trial, many have advocated for testimony about post-traumatic stress disorder (PTSD) instead. One reason why the use of this diagnosis proves to be more helpful in cases which a vic

defense; PTSD is better established as a mental disorder

One of the most criticized parts of the battered woman syndrome (BWS) is the _____ component.

learned helplessness

Which one of the following syndromes is an established medical condition listed in the Diagnostic Statistical Manual Disorders (DSM-5)?

post-traumatic stress disorder

Studies have found that women who used violence against their abusers were likely to be found ______ in a jury trial, and expert testimony on BWS was likely to _____.

guilty; make little difference

Although a battered woman's reluctance to leave the abusive relationship ______, research has found that a lot of battered women are killed by their abusers _____.

may seem illogical; after leaving

According to Burgess and Holmstorm (1979) who coined the term RTS, _______ of rape victims report that they still have not recovered 5 years after the incident.

about a quarter

Only 200 years ago, women were treated like _____ and most wife beatings were seen as a ____ matter.

property; private

The concept that individuals with impaired cognitive understanding should not be held culpable, was used as early as the Roman Empire. Roman law classified those individuals to be non compos mentis, or:

without mastery of mind

Almost half of the U.S states allow for a guilty but mentally ill (GBMI) verdict. Mnay scholars question the GBMI verdict since it does not guarantee that the offender will:

receive effective mental health treatment

______ is the concept that the punishment of criminals teaches others that criminal activities lead to penalties.

general deterrence

In the reconstruction of memory, it is not uncommon to find that distortions may take place at any stage. The following statement about this process is INCORRECT:

Various sounds encoded during memory formation are the easiest part to receive.

When detectives were asked in a study about their views on profiling, most said they found it:

helpful

Two components of the presumed child sexual abuse accommodation syndrome (CSAAS) are the denial of abuse and recantation of abuse. Researchers who analyzed actual interviews with sexually abused children found that, in reality, the denial and recantation

are unusual.

The memory process is often describe as having the following components:

encoding, storage, retrieval.

Cross-race-effect has been shown to maintain consistent strength:

for people of different ages.

When describing memory, ______ is the first component required in the process necessary for retention.

encoding

The benefit of the cognitive interview is that:

it helps the witness relax and mentally reinstate the details of the crime event.

When an eyewitness is a child, it is likely that the accuracy of information the child can provide is _____ that provided by an adult.

lower compared to

Research is an important part of understanding how eyewitness accounts are derived. As such, a range of factors are often considered. Such factors as the procedures for constructing lineups and questioning witnesses are called:

system variables.

Research has shown that children _____ sometimes have difficulty distinguishing between imagined and real events.

under 5

We should be especially skeptical of allegedly recovered memories in the following cases, EXCEPT:

memories of abuse came back as flashes of particular incidents.

According to the text, the most common response to traumatic experiences is not _____ but _____.

forgetting; flashbacks

According to the text, children who recant previous allegation of sexual abuse are likely to have been abused by a:

parental figure.

in some cases so-called memories of past abuse have surf by _____ of the alleged victim while under hypnosis or guided imagery.

suggestibility

Which of the following events have prompted psychologists to conduct additional research on and develop new techniques for eliciting accurate reports of abuse?

Multiple reports of sexual abuse of children in day care centers.

Which of the following events prompted psychologists to conduct additional research on and develop new techniques for eliciting accurate reports of abuse?

Multiple reports of sexual abuse of children in day care centers.

Bearing witness to what someone else said outside of court is called:

hearsay testimony

Bearing witness to what someone else outside of court is called:

hearsay testimony

Madame Popova, who confessed to involvement in the murders of over 300 men in Russia in the 1800's, is an example of what type of serial killer?

murder for profit

All of the following statements are true about implicit bias, EXCEPT:

implicit bias cannot be corrected, even with time and reflection.

Courts can recommend forcible medication of defendants:

when they are a danger to self or others.

Competency to stand trial (CST), according to the Dusky Standard, has two basic components to it's definition. The first component refers to the accused individual's ______. The second component refers to the accused individual's _____.

ability to interact rationally with an attorney; understanding of how the court process works

There are several important reasons why the defendant should be able to understand what is happening at every stage in the criminal justice process. Which of the following is NOT such a reason?

It is easier for the prosecutor to convict the defendant who lacks the mental capacity to understand the basics of a legal proceeding.

Prior to Jackson v. Indiana (1972), all of the following were true about the defendants that were deemed incompetent, EXCEPT:

they stayed in mental asylums no longer than their expected prison sentence.

If restoration to competence cannot be accomplished, proceedings may be held to issue a(n) _____ commitment.

involuntary civil

After a federal judge ruled that Jared Loughner was incompetent to stand trial, he was:

sent to a medical facility to have his competency restored.

Most states ______ have a formal process for the certification of CST evaluators, ______ specific test to determine competence.

do not; and there is no

Jared Loughner was diagnosed with schizophrenia, which is a mental disease that can often lead to the defendant's competency being questioned and a need for competency evaluation. According to the text, people with schizophrenia exhibit the following psyc

collusion

What is a primary advantage of providing an inpatient rather than an outpatient evaluation of competency?

It provides multiple opportunities to observe the defendant's behavior over time.

When determining insanity, the courts rely on:

clinical psychologists

The principle of punishment known as retribution is embodied by the following saying:

An eye for an eye

the concept that a person commits a crime, receives punishment for it through the workings of our legal system, and afterwards makes a decision to never commit a crime again to avoid such punishment, is called:

deterrence.

Unlike with insanity defense, mens rea defense requires that the _____ must prove mens rea ______.

prosecuting attorney; beyond a reasonable doubt

For the most part, changes in insanity law result from the public's reaction to:

the outcomes of high-profile cases

Research first completed by Rita Simon in 1967, and later replicated in multiple studies by Finkel and his colleagues in the 1900s and 2000s, found that, when different sets of jurors are given different definitions of insanity based on legal terminology,

no impact on their verdicts

The Durham standard replaced the M'Naghten rule in 1954, to take into consideration:

whether the act was caused by the mental disease or defect of a person.

A sleepwalking defense is more likely to be successful when ______.

previous sleepwalking is documented.

Survey estimates cited in the text place the percentage of women who have ever experienced intimate partner violence at over _____. A corresponding figure for men is over _____.

30%; 20%

Betty endured abuse throughout most of her 30+ years of marriage. During that time, she became less resistive to the attacks and more submissive. A term Lenore Walker used to describe this state is:

learned helplessness

Marissa has been beaten by her husband on multiple occasions. She has learned to anticipate her husband's impending violence by carefully observing minute signs of his moods and behaviors. Which of the following terms is used to describe this type of heig

Hypervigilance

The use of BWS in courts _____ BWS has been widely ______ by the scientific community.

continues unbated though; criticized and seen as controversial

According to the text, men in intimate relationships are ______ the victims of physical assault.

sometimes

Regarding the scientific validity of BWS, all of the following statements are correct, EXCEPT:

There has been a great deal of follow-up research to support the validity

Courts have been more receptive to clinical psychologists than to other types of psychologists. The most probable reason is that, like lawyers, clinicians:

are oriented toward the individual case

Some experts argue that, battered women cases, expert testimony should focus on the social reality of the woman's situation rather than on her psychological reactions. Which of the following is NOT a question that such social agency framework (SAF) testim

How charming and charismatic was the abuser to entice the woman to stay with him?

The similarity-leniency hypothesis seems to generally apply when _____. In such cases, jurors tend to ____ to those defendants who are like them.

evidence is weak or inconclusive; give the benefit of the doubt

Studies show that juror characteristics generally have a _____ impact on the verdict, and thus the idea that picking a jury makes or breaks the case seems _____ as a general rule.

weak; overstated

The one day or one trial system is used to:

reduce the number of hardship exemptions.

Mock juries benefit trial lawyers when the actual trial and jury selection takes place because:

profiles developed through the mock jury studies guide the choices for the real case.

Based on the text, the advantage of information-gathering techniques over accusatorial methods is that:

both approaches increase the number of true confessions while information-gathering techniques reduce fake confessions.

To make jury service less burdensome, the one day or one trial system is used in many areas. It means that:

for jurors who are selected, the jury duty will be over after the trial ends, and for the dismissed jurors, the jury duty is over.

In terms of "burden of proof", in criminal cases, the standard of proof for a person to be found guilty is__________ whereas in a civil trail, the standard of proof is usually __________.

beyond a reasonable doubt; preponderance of the evidence

A challenge for cause _______, whereas a peremptory challenge _______.

is based on possible biasing factors; allows lawyers to dismiss a person with no reason given.

Information that might be deemed prejudicial is likely to be considered as:

inadmissible evidence

According the story model, jurors construct their stories _____ the evidence at trial.

while hearing

Evidence that is meant to damage the credibility of a witness or the defendant's reputation is also referred to as ______ evidence.

impeachment

According to the text, it is useful to think of a trial as a _____ to help a jury or judge ______.

method of providing information; arrive at a decision in the case

Studies show that a judge's instruction to ignore certain information as inadmissible evidence may often lead to the opposite: the effect of this information on the jurors is amplified. the text mentions all of the following explanations as possible reaso

Whether jurors give weight to the evidence depends on their knowledge of the defendant.

Members of a jury that appear to have a great deal of influence on the deliberation process are referred to as _____ jurors.

strong

In the case of a split jury, where about half favor "guilty" and the other half favor "not guilty" on the initial vote, it is more likely that the final verdict will be _____. This tendency is also referred to as the _____ bias.

not guilty", leniency

Liberation hypothesis postulates that in situation where evidence is _____, jurors base their decisions on other factors such as their _______.

ambiguous; past evidence or prejudice

When it comes to child custody decisions, Texas is the only state where:

juries decide custody arrangements

One of the main reasons for ethics board complaints and malpractices suits against psychologists is:

child custody evaluations

A process alternative to litigation is mandatory in many states for couples seeking divorce. A neutral third party who approaches the parents together in non-adversarial manner is called a:

mediator

When courts make decisions about child custody, they are supposed to take into account all of the following EXCEPT for the:

parent's sexual orientation

Matthew's parents divorced after a tumultuous marriage. It was difficult for him to understand what happened because he has become somewhat accustomed to all of the frighting. In addition, the divorce resulted in a difficult financial situation, and he ha

resilience

Although the tender years doctrine has been _____, most court decisions about child custody award ______.

abandoned; custody to the mother

One serious problem with the BICS is that it can:

escalate conflicts between parents.

The best interest of the child standard may inadvertently increase litigation between parents. One way to minimize this problem is to adopt:

preferred custody arrangements

More often than not, children define family in terms of:

emotional ties

Because clinicians were not aware of how often offenders reoffend, which is called the ______, clinicians' predictions would _______ the likelihood of violent behavior.

base rate of reoffense; overestimate

In Kansas v. Crane (2002), the Supreme Court held that the following conditions are required for involuntary civil commitment under SVP laws, EXCEPT:

a history of violent behavior.

When it comes to sex offender community notification and registration laws, there is ______ scientific evidence that they reduce sexual recidivism. Most critics argue that these laws ______.

no; only make the public feel safe without actually affecting safety

When actuarial assessment of violence risk is conducted, all of the following factors would be classified as historical markers, EXCEPT:

persistent strong feelings of anger.

There are two aspects related to the purpose of the sexually violent predator civil commitment laws (SVP laws). One is the state's authority to protect the citizenry, also referred to as _____ power. The second component is the state's duty to protect tho

police; parens patriae

Threat/control-override symptoms are common in people suffering from _____. What leads to the symptoms is their common delusion that others _____ them.

paranoid schizophrenia; want to harm

Cathy was incarcerated because she has a history of physical and mental abuse of her stepchildren. She has been held in custody for an extended period after her incarceration term ended because she has the possibility of becoming violent, which is also kn

preventative detention

Among dynamic markers of risk in actuarial prediction of future violence, there are all of the following factors, EXCEPT:

antisocial personality disorder

Community notification requiring states to make personal and private information about known sex offenders available to the public is a result of the enactment of the:

Megan's Law.

The Supreme Court case of Kansas v Hendricks (1997) has addressed several challenges about potential constitutional violations of SVP laws. Which of the following is NOT among these challenges?

SVP laws deprive individuals of their constitutional right to bear arms.

Most hate crimes in the United States over the last three years have targeted victims based on their _____.

race or ethnicity.

Which of the following innocent people was NOT executed?

Kirk Bloodsworth

A defendant is deemed "guilty" or "not guilty" during ______ of a capital murder trial.

phase one

Three of the four forms of execution below are only used in the United States. Which one is used in other countries that administer the death penalty?

Hanging

Compared to jurors who are screened out by the death qualification process, death-qualified jurors possess all of the following characteristics, EXCEPT:

they are more biased against the victim

During the process of death qualification, if a potential juror ______, he or she is excluded from serving on a capital jury.

is unwilling to consider execution as a punishment.

In the parts of the world that still recognize the death penalty, the methods of execution often include hanging and shooting, as well as:

beheading

Research by Lynch and Haney (2000), using identical cases but varying the race of the victim and the offender, has found the following: when the defendant was black and the victim was white, ______ percentage of mock jurors recommended the sentence of dea

a higher; did not understand the instructions well

Part of the reason for the Supreme Court decision in Roper v Simmons (2005) was a result of research findings showing that key psychological capacities of juveniles are not fully developed compared to adults. These key capacities include all of the follow

emotional expression

The definition of aggravated murder punishable by death _____ across jurisdictions

varies

More than 2,900 individuals are now on death row in the United States. It os most likely that the majority of them will:

die of natural causes.

During the 1960s-1980s, polygraph use was:

a lucrative business

According to the text, confirmation bias is one's tendency to:

seek out evidence that supports one's pre-existing beliefs

The comparison question test (CQT) posits that guilty individuals react more strongly to ______ questions, whereas innocent individuals react more strongly to _____ questions.

relevant; comparison

According to the text, EEG reads neural impulses ______, whereas fMRI reads brain activity ______.

continuously; every 1-5 seconds

Unlike other polygraph tests, the guilty knowledge test (GKT):

reduces polygraph examiners to mere technicians

Since the 1960s and throughout the present time, an interrogator in the United States would be likely to use all of the following forms of coercion to elicit confession, EXCEPT:

promising lenient sentencing

When a confession exists, juries ________ come back with a conviction.

very often

Research in the field of social psychology has discovered that people tend to _______ the impact of situational forces, and this process is responsible for believing that suspects in police custody would _____ to a crime they did not commit

disregard; not confess

Utilization of a team approach to interrogation often takes the form of a _____ approach.

good cop-bad cop

Jeremy has worked in a prison for 15 years and has been called to jury duty. He is to serve on a criminal case involving the shooting of a guard during a prison riot. Based on the definition of challenge for cause, Jeremy would be a(n):

unlikely juror.

The tendency to hold an individual's dispositional causes such as personality responsible for his or her behavior is referred to as the:

fundamental attribution error

According to the text, legal culture tries to avoid _____, whereas for psychology, ______ is an integral part of the scientific process

uncertainty

Between the 1930s and 1960s, an interrogator in the United States would be likely to use all of the following forms of coercion to elicit confessions, EXCEPT:

direct physical violence

Which one of the following is NOT likely to lead to a false confession?

Suspect's confidence

Santiago is a clinical psychologist. His knowledge and skills might be most relevant for the legal system in which one of these cases?

When assessing a mentally ill defendant's competency to stand trial

Self-defense can be used in a court of law when a women is facing charges as a result of killing her batterer. However, specific criteria need to be met to plead self-defense. Which of the following does NOT meet the requirements of self-defense?

The abuse was regular, and the victim felt scared even when the abuser was asleep

The famous Brown v. Board of Education (1954) case, besides being one of the most important Supreme Courts rulings of the 20th century, was a milestone in the psychology and law alliance for the following reason:

the ruling cited research conducted by social scientists.

In consideration of scientific testimony, gatekeeping in the legal sense includes:

assessing the scientific validity of potential testimony.

All of the following are possible explanations for why suspects waive their Miranda rights, EXCEPT:

police are very careful in explaining to suspects their rights.

According to a study conducted by Honts and colleagues (1994), the use of countermeasures _____ the detection of guilty suspects by 50%, and the examiners ____ able to tell that the suspects were manipulating their own arousal patterns.

reduced; were not

One of the most important tasks of the polygraph examiner using the standard procedures like CQT is to:

convince the examinee that the polygraph can always detect a lie.

Reliability refers to:

the consistency of a measure or observation.

Shelly testifies in court that the DNA recovered at the crime scene is likely to come from the suspect because less than 0.0001% of population have this specific genetic marker. This type of identification is called a(n):

match plus statistics.

Two fingerprint examiners have independently concluded that the suspect can be excluded as the source of the print. This scenario illustrates the _____ of fingerprint analysis as a measure.

interrater reliability.

According to the text, forensic experts testifying in court often cannot provide scientific data about error rates. Instead, they are more likely to invoke their:

vast experience as a fingerprint examiner.

A shoe imprint left at the crime scene will be much more helpful if it comes from a(n) _____ shoe.

well-worn

A qualitative statement made by an expert when presenting evidence is:

a non-statistical statement about the strength of a match.

Diego is a forensic expert who is asked to testify whether the wound that the victim died from came from a knife found in the suspect's car. Diego has confirmed a high level of agreement between the knife and the wound, which in his professional opinion m

individualization

When comparing the latent prints with the suspect's fingerprints, a _____ makes a decision on whether there is a _____.

person; match

Which of the following is a correct statement about research on profiling?

There has been little systematic research on profiling.

Research has essentially discredited the idea at the heart of the profiling process; that _____ crimes are committed by _____ people

similar; smiliar

In a study examining the effectiveness and accuracy of profiling, the vast majority of criminal profiles were found to be:

inaccurate and inconsistent.

According to the text, one of the first criminal profiles was developed by Dr. Bond based on the victim's autopsies in the _____ case, in ______.

Jack the Ripper; 1888

Psychological profiling mostly relies on the _____ of a particular profiler.

skilled intuition

Correll and associates asked various groups of participants to engage in a First Person Shooter simulation. In the simulation, participants saw unarmed and armed targets, and were supposed to shoot the armed targets and not shoot the unarmed targets. Some

all of these statements are true

Female serial killers are more likely than male serial killers to kill

by poisoning

The concept of repression has found _____ support in empirical research studies of real victims of ______.

very little; traumatic events

When NICHD Investigative Interview Protocol procedures are followed, interviews with children are ______ recorded on video.

almost always

The concept of repression was popularized by:

Sigmund Freud.

According to research noted in the text, jurors that heard both adult hearsay witnesses and child testimony deemed the hearsay testimony:

more credible.

Research has demonstrated that some techniques used in interviewing children in abuse cases are especially helpful. These techniques include all of the following, EXCEPT:

asking the same question more than once.

Elizabeth Loftus is well known for her research on false memories. In one experiment, she created a false memory in participants of being ______. Of the participants in the experiment, ______ have "remembered" most or all of the implanted event after two

lost at the mall; 25%

______ is the term used to describe the biochemical representation of our experiences within the brain, which are known to deteriorate over time.

Memory trace

The Manson criteria, emphasized by courts, consist of five factors to be taken into account when evaluating eyewitness identification accuracy. Which criterion is the most problematic one, according to the text?

The witness's degree of certainty about her memory of the event.

The term _____ is part of memory processing. It means gathering information and putting it into a form that can be used later on.

encoding

Ashley stopped at the bank to deposit her paycheck, and the stopped at a nearby gas station to pick up a few grocery items. While she was at the gas station, a robber entered with his gun drawn and ordered everybody, including Ashley, to raise their hands

unconscious transference

The biggest problem regarding the use of the cognitive interview with eyewitnesses is the difficulties that police have with:

adopting the interviewing style that is so different from their usual one.

Even though the Supreme Court _____ the Manson framework, several states continue to ______ criteria established in Manson.

refused to amend; reexamine the outdated

Oftentimes, the post-identification boost in an eyewitness's confidence level about his or her testimony can be explained by the cognitive dissonance, which is a(n):

uncomfortable feeling when someone has conflicting thoughts.

When performing competency evaluations within an institution, collateral sources of information can be gathered regarding the defendant's behavior. Some of the individuals that might be able to provide this additional information include:

prison guards, nurses, and mental health professionals.

An evaluation of competency can be requested or ordered any time during the trial is there is a _______ doubt about the defendant's competency. This type of doubt is a(n) _______ the preponderance of evidence standard.

bona fide; even lower standard than

Adjudicative competence is a legal concept that describes the defendant's ability to:

participate effectively in legal proceedings.

Preponderance of the evidence standard of proof applies to the competency to stand trial, just as it does in many types of court cases. In the case of competency to stand trial, the defense must prove that:

it is more likely than not that the accused is incompetent.

Ester was born and raised in Iran, and her knowledge of an adversarial criminal justice system is very limited. According to the current legal standards in the United States, her competency to stand trial would only be affected is she:

thinks her attorney came from another planet.

Competency to stand trial is _______ proven _______.

presumed unless; otherwise by the defense

A recent study by Gowensmith and colleagues (2012) that analyzed competency cases in Hawaii, found that three different independent evaluators agreed on whether the defendant was competent or incompetent about _____ of the time, and that judges agreed wit

70%; over 90%

According to the text, insanity:

refers to the criminal's state of mind at the time the crime was committed.

A crucial question at the center of Andrea Yates trial was whether:

she knew the difference between right and wrong.

The opposite to the Roman concept of non compos mentis is an awareness that criminal behavior is wrong. In the modern legal system, a term for such awareness is:

mens rea.

On several occasions, Andrea Yates tried to kill _____, and still the jury at her first trial found her ______.

herself; guilty

Research has shown that sleepwalkers

can perform complex activities such as driving a car while asleep.

After enduring years of abuse including regular beating and humiliation from her alcoholic husband, Antoinette killed him one day after he fell asleep on the couch in a drunken haze. Now Antoinette is on trial for her husband's murder. The most likely ver

guilty

Judicial nullification instruction can best be described as an instruction that lets jurors know that they can _____ strict interpretation of the law _____ that legal interpretation would result in an unjust verdict.

disregard; if

Victims of rape trauma syndrome (RTS) and battered women syndrome (BWS) may also meet the criteria from ______, which is an establish medical diagnosis.

posttraumatic stress disorder

A common term used to describe a group of associated symptoms that lead to a significant dysfunction in the performance of normal activities is a:

syndrome.

According to the movie "The Burning Bed," Francine Hughes finally began to cooperate fully with her lawyer because _______.

she received a letter from another battered women who had read about her case.

The ______ hypothesis predicts that, when the characteristics of jurors and of the defendant are similar enough, it might influence verdicts.

similarity-leniency

Negative pretrial publicity makes jurors more likely to see the defendant as ______. When studies assessed if the judge's instructions to disregard to the negative publicity make a difference, they found that such instructions _______.

guilty; do not remedy the situation

Automatic exemptions for such things as not understanding English well or being a police officer:

are being reduced in recent years.

With regard to strategies for jury selection, social scientists and lawyers appear to have different approaches, which became especially evident in the O. J. Simpson case. Social scientists rely on ______, whereas lawyers rely on _____.

data collection; intuition

Among the traits noted as being associated with jurors' verdicts is how people tend to explain what happens to them, or how they answer the question: Do you believe that what happens to you is usually the result of your own behavior? This trait is referre

locus of control.

_______ attorneys get more peremptory challenges because ______ at the trial.

Defense; their clients have more to lose

Because of a long, exhausting interrogation, Kathryn just wanted the questioning to stop, so she confessed to the crime knowing she did not commit it. This type of confession is called an:

instrumental-coerced confession

Daniela was ______ and she waived her Miranda rights during her arrest. She decided to tell the police her side of the story without waiting for an attorney. The most likely reason she did that is because she has:

innocent; nothing to hide

Some promising low-tech methods of lie detection include all of the following, EXCEPT:

making suspects listen to their own story recorded

The main similarity between Bertillon's anthropometry and modern biometrics is that both:

aim to identify an individual based on measurable anatomical traces.

The Combined DNA Index System (CODIS) maintained by the FBI:

is the most frequently used DNA database in the world.

Which one of the following situations would be an example of a false negative error?

An expert declares there is no match when it actually matches.

To reduce error and bias, a blind test can be administered to a forensic examiner, which means that the examiner:

does not know he or she is being tested.

According to the text, an attempt to examine the mental state of an individual prior to his or her death is known as:

psychological autopsy.

According to the text, Dr. ______ is considered to be the first individual to develop a criminal profile when working on the Jack the Ripper case, at the end of the 19th century.

Thomas Bond

The hypothesized differences between organized and disorganized killers:

were not confirmed in empirical studies.

Killers that can be describes as carefully selecting their victims are referred to as:

organized killers.

Alexis is trying to figure out whether murders in three different states were committed by the same person. This process is referred to as:

case linkage.

According to the text, courts have been reluctant to allow expert testimony based on psychological autopsies. One exception to this rule is a routine use of psychological autopsies in cases where:

the will is contested based whether the deceased person legally competent when the will was written.

Holmes and Holmes (2010) have categorized serial killers into four types: visionary, mission-oriented, hedonistic, and power-oriented. Subsequent research showed that:

most serial killers do not fall neatly into just one of these categories.

Much research has demonstrated that most victims of traumatic events have ______ memories of the episode.

vivid

When exposed to sexual abuse testimony given by children at trial, it appears that jurors are _____ to believe young children compared to adolescents.

more likely

In the mid-1990s, the hysteria of day care child abuse cases has ______ while claims of having recovered memories of past sexual abuse have ______.

basically died down; proliferated

Suggestive questions can be best described as _______ leading questions ________.

including; not volunteered by the child

Lapses of attention are most likely to affect which stage of the process of memory work?

Encoding

According to a study completed by Morgan and his colleagues (2004), stress appears to have an effect on eyewitness identification. Based on the study results, the rate of correct identification was ______ in low-stress conditions.

higher

According to the text, what we generally see and remember is impacted by what we expect to see. This tendency to maintain firmly held beliefs about a sequence of expected behaviors is referred to as:

preexisting scripts.

The facts that eyewitness testimony is so powerful in influencing jurors is only a problem if:

the witness is mistaken.

Certain ways of initial questioning of the eyewitness can significantly alter his recall of the crime scene. Retrieval inhibition occurs when an individual:

selectively recalls some aspects of a situation while other aspects get ignored.

When doing research on eyewitness memory, social scientists often look at many factors. Factors that are outside the control of the legal system are referred to as:

estimator variables

Peter was able to pick a guy who looked like his assailant out of a ________. When he was told that this was very helpful to the detectives working the case, Peter felt ______ certain that he remembered his attacker well.

lineup; more

Grisso and colleagues (2003) compared youths (ages 11 to 13) with young adults and found that youths are _____ likely to accept a plea bargain.

more

According to research completed by Grisso and his colleagues (2003), the differences between adolescents (ages 11 to 17) and young adults (ages 18 to 24) in terms of their functioning on CST-related abilities were:

significant.

The psychiatric illness most commonly associated with successful insanity please include all of the following, EXCEPT:

anxiety disorders

A sleepwalking defense is more likely to be successful when _____

previous sleepwalking is documented.

The text indicates that one possible application of a useful batterer typology is to:

develop targeted interventions.

Burgess and Holmstorm (1979) have described recovery from rape as a two-stage process consisting of the ______ phase and the _____ phase.

acute crisis; reorganization

Chris is extremely jealous and has a fear of abandonment that often spills into impulsive violent altercations with his girlfriend, especially if she spends any time with her male friends. On the other hand, he appears totally normal and nonviolent to his

borderline personality disorder.

According to the text, most rapes are committed by ______, with only approximately 15% of rapes being committed by ______.

acquaintances; strangers

It is often asserted that the majority of Americans ___ the death penalty. However, if people are asked to compare the death penalty with an alternative of like imprisonment, most respondents chose _____ as a more acceptable punishment.

support; life imprisonment

Capital crimes under the federal law include all of the following crimes, EXCEPT:

possessing bomb-making material and instructions

Different factors come into play during the sentencing phase of a homicide case when it is decided whether the sentence should be death or life in prison with no hope of parole. Circumstances that support the death sentence are called:

aggravating factors

Excluding federal crimes, the only crime punishable by death in the United States is:

aggravated murder

Research on possible errors in death penalty cases has examined every capital case in the United States over a 22-year period. It was found that 68% of death sentences reversed because of serious errors at trial. All of the following were found to be the

incompetent judges or juries

Interestingly, research shows that, when mock jurors understand the instructions, there is no difference in the rate of death penalty recommendations based on race or ethnicity. However, when jurors do not fully comprehend the instructions, there is a cle

black defendants

The process of death qualification excludes about a quarter to a third of potential jurors from serving on capital juries. The excluded jurors are more likely to possess the following characterists, EXCEPT
:

tendency to value law and order

The term "Prejudice" is used to refer to a(n) ______ while the term "discrimination" refers to a(n) ________

attitude, behavior

In Barefoot v. Estelle (1983), the American Psychiatric Association provided an amicus curiae brief stating that expert predictions of violence risk were _____ two out of three times. The Supreme Court held that experts ______ provide their opinion in suc

incorrect; should still

The Supreme Court made it clear in Barefoot v. Estelle (1983) that expert predictions of future dangerousness were ______ in capital sentencing _______.

admissible; despite their likely inaccuracy

According to a report completed by Gookin (2007), nearly _______ have been incarcerated as a result of the SVP laws in the United States sine 1990.As of 2007, _______ of them have been released.

4,500 sex offenders; 10%

All of the following statements are true about actuarial risk assessment, EXCEPT:

research shows that clinicians can improve the accuracy of actuarial prediction if they adjust it based on their clinical intuition

Until the second half of the twentieth century, wife battering was treated ____ most other forms of violence.

much less seriously than

Statistics noted in the text indicate that serious violence perpetrated by intimate partners is _____ to happen against women.

more likely

The primary caretaker rule takes into account several factors when determining the appropriate custody arrangement. Which of the following factors is NOT generally considered?

Who earns the money paid for the child's daycare or school.

Certain characteristics of the defendant are often considered by jurors. For example, Phillip is a known gang member. He was on trial for the death of a priest that he ran over will evading the police. In this case, it is likely that Phillip will receive

harsher than; fellow gang member

When jurors are instructed to disregard the inadmissible evidence, they sometimes feel there is no good reason for such an instruction. Instead, they hive that evidence an even greater weight in their decision making. This process is assumed by ______ the

reactance

Unanimity is required by ______ of the states in capital murder trials.

all

Projective tests like the Rorschach inkblot test measure an individual's response to _____ stimuli. The continued use of these tests is ______ scientific validity.

ambiguous; unadvisable due to their lack of

When there is a history of spousal abuse, the abuser may use the mediation process to have access to the victim. In the case, mediation:

may not be the best approach.

According to the text, all of the following statements about people with serious mental illnesses are correct, EXCEPT:

Playing video games increases the likelihood of violence.

At its high point (in the 1950s), the number of people housed in ____ in the United states reached close to ____ individuals

mental asylums; half a million

In cases of divorce under English common law, children were considered to be:

property of the father

Considering the child first and foremost in a custody decision has been reinforced by the establishment of the:

best interest of the child standard.

Which of the following statements reflects the current state of knowledge about parental alienation syndrome (PAS)?

It has not been confirmed scientifically because there is not enough research data to support its existence.

Information released in the media through pretrial publicity is often ____ at trial.

inadmissible

The foreperson of a jury is regarded as its leader. However, a foreperson may or may not exert disproportionate influence on a verdict. In fact, he or she may become more focused on the _____ and on acting as a(n) _____.

procedural issues; moderator

According to extensive research, the best predictor of what the verdict will be is the ______ of relevant evidence.

strength

Juries have the power to make all of the following life-changing decisions, EXCEPT:

pardon a death-row inmate.

When jurors change their votes under the pressure from other jurors, even though they did not change their minds, this phenomenon is also referred to as:

normative influence.

What is the ethical question most pertinent to use of jury consultants?

Do jury consultants make it more likely for a wealthy defendant to win a case?

What is usually called jury _____ is actually a process of _____.

selection; eliminating candidates from the jury pool

The size of venire depends on all of the following, EXCEPT:

whether the defendant is indigent.

According to the text, all of the following categories of people are significantly more likely to be found incompetent, EXCEPT:

youths with physical disabilities.

The Insanity Defense Reform Act (IRDA) of 1984 required that the _____ prove the insanity of the defendant at the time time of the crime _____.

defense; by clear and convincing evidence